-2
$\begingroup$

Let $\{a_k\}_{k\geq 1}$ be a sequence of positive integers such that each $a_k > 1$. Show that every real number $x\in[0,1)$ can be represented as $$x=\sum_{k=1}^\infty\frac{x_k}{a_1a_2 \cdots a_k} ,$$ where $x_k\in\{0,1,...,a_k-1\}$.

Edit: This is part of Theorem 1.6 in Ivan Niven's Irrational numbers.

When $a_k = 10$ for all $k$, this becomes the well-known fact that every number in base-$10$ expansion $0.x_1x_2x_3x_4\cdots$.

|cite|edit|reopen|undelete (1)|flag
$\endgroup$

deleted by user21820, José Carlos Santos, RRL Apr 1 at 17:06

closed as off-topic by user21820, José Carlos Santos, Saad, GNUSupporter 8964民主女神 地下教會, Xander Henderson Mar 26 at 14:12

This question appears to be off-topic. The users who voted to close gave this specific reason:

  • "This question is missing context or other details: Please provide additional context, which ideally explains why the question is relevant to you and our community. Some forms of context include: background and motivation, relevant definitions, source, possible strategies, your current progress, why the question is interesting or important, etc." – user21820, José Carlos Santos, Saad, GNUSupporter 8964民主女神 地下教會, Xander Henderson
If this question can be reworded to fit the rules in the help center, please edit the question.

2
$\begingroup$

Choose $n_1=\lfloor a_1x\rfloor\in[0,a_1-1]$ so that $x_2=a_1x-n_1=a_1x-\lfloor a_1x\rfloor\in [0,1)$. Construct inductively $n_k$ and $x_{k+1}$ by $$ n_k=\lfloor a_k x_k\rfloor,\quad x_{k+1}=a_kx_k-n_k\in [0,1). $$ Then we can prove inductively that $$ (*)\ : \ x-\frac{n_1}{a_1}-\frac{n_2}{a_1a_2}-\cdots-\frac{n_k}{a_1a_2\cdots a_k}=\frac{x_{k+1}}{a_1a_2\cdots a_k}\in [0,\frac1{a_1\cdots a_k})\subset [0,2^{-k}). $$ That is, assuming $(*)$ is true for $k$, we have $$\begin{align*} x-\frac{n_1}{a_1}-\frac{n_2}{a_1a_2}-\cdots-\frac{n_k}{a_1a_2\cdots a_k}-\frac{n_{k+1}}{a_1a_2\cdots a_{k+1}}&=\frac{x_{k+1}}{a_1a_2\cdots a_k}-\frac{n_{k+1}}{a_1a_2\cdots a_{k+1}}\\&=\frac{a_{k+1}x_{k+1}-n_{k+1}}{a_1a_2\cdots a_{k+1}}\\&=\frac{x_{k+2}}{a_1a_2\cdots a_{k+1}} \end{align*}$$ holds for $k+1$. It follows $$ x-\sum_{k=1}^N\frac{n_k}{a_1a_2\cdots a_{k-1}a_k}\xrightarrow{N\to\infty} 0. $$

|cite|edit|flag
deleted Apr 1 at 17:06
$\endgroup$

Not the answer you're looking for? Browse other questions tagged or ask your own question.